Difference between revisions of "2019 Mock AMC 10B Problems/Problem 3"
(One intermediate revision by one other user not shown) | |||
Line 1: | Line 1: | ||
After trying each option we have | After trying each option we have | ||
− | <cmath></cmath>A) <math>3^ | + | <cmath></cmath>A) <math>3^\frac{2018}{3}</math> which is irrational as 2018 is not divisible by 3 |
− | <cmath></cmath>B) < | + | <cmath></cmath>B) <math>3^\frac{2019}{2}</math> which is irrational as 2019 isn't divisible by 2 |
− | <cmath></cmath>C) <math>3^2+\sqrt{2}^2+6\sqrt{2}</math> which equals <math>11+6\sqrt{2}</math> which is irrational | + | <cmath></cmath>C) <math>3^2+\sqrt{2}^2+6\sqrt{2}</math> which equals <math>11+6\sqrt{2}</math> which is irrational |
− | <cmath></cmath>D) < | + | <cmath></cmath>D) <math>(2pi)^2</math> equals <math>4pi^2</math>, which is irrational |
− | <cmath></cmath>E) <math>(3-\sqrt{2})(3+\sqrt{2})=9-2=7</math> which is rational | + | <cmath></cmath>E) <math>(3-\sqrt{2})(3+\sqrt{2})=9-2=7</math> which is rational |
− | We have < | + | We have <math>\boxed{\bold{E}}</math> <math>(3-\sqrt{2})(3+\sqrt{2})</math> |
Latest revision as of 10:47, 5 November 2019
After trying each option we have A) which is irrational as 2018 is not divisible by 3 B) which is irrational as 2019 isn't divisible by 2 C) which equals which is irrational D) equals , which is irrational E) which is rational We have